Eine intuitivere Lösung für die Kinematik des Rollen-Seil-Systems

Hier ist ein kinematisches Problem, das ich mit einem streng geometrischen Ansatz gelöst habe. Ein Gymnasiast schlug jedoch einen anderen schnelleren Ansatz vor, der seine eigenen Feinheiten hat. Ich möchte verstehen, wie man den letzteren, schnelleren "High-School" -Ansatz richtig anwendet.

Das Problem

Wir haben diese kinematische Situation, wo die 2 Seile mit einer Geschwindigkeit u nach unten gezogen werden . Der Block bewegt sich mit der Geschwindigkeit v nach oben . Wir müssen die Beziehung zwischen u und v finden .

Geben Sie hier die Bildbeschreibung ein

Die richtige Antwort auf das Problem lautet:

v = u C Ö S ( θ )

Zweifel Nr. 1: Der schnellere High-School-Ansatz – Wie funktioniert er?

Geben Sie hier die Bildbeschreibung ein

In diesem Diagramm schlägt der Schüler vor, zuerst eine Senkrechte von A nach OB auf C fallen zu lassen. Jetzt rein Δ A B C , wir können das sehen u = v C Ö S ( θ ) .

Die Grundlage für dieses Argument ist jedoch nicht klar. Ich meine, ein anderer Student kam mit einer anderen Antwort, als er das sagte, wenn wir uns das ansehen Δ Ö A B , wir sehen das v = u C Ö S ( θ ) .

Wie genau funktioniert dieser Ansatz? Woher wissen wir, welche Antwort die richtige ist?

Zweifel Nr. 2: Warum summieren sich die kinematischen Größen nicht so wie die dynamischen Größen?

Ein anderer Student äußerte einen weiteren interessanten Zweifel. Die Antwort, auf die er kam, lautete:

v = 2 u C Ö S ( θ )

Sie müssen erraten haben, warum er mit dieser Antwort kam. Sein Argument war, dass, da es zwei symmetrische Seile gibt, ihre Bewegung sich addieren wird, um die Bewegung des Blocks zu ergeben. Er sagte, so wie sich Kräfte addieren, sollten sich auch die Verschiebungen/Geschwindigkeiten addieren. Nun, das Argument ist eindeutig trügerisch.

Aber wie erkläre ich einem Gymnasiasten den Trugschluss?

ANHANG – Der rigorose geometrische Ansatz:

Hier ist der formale geometrische Ansatz, den ich verwendet habe, um die richtige Antwort abzuleiten.

Geben Sie hier die Bildbeschreibung ein

In dem oben gezeigten Diagramm wissen wir, dass:

P Ö B = L  (Konstante, Gesamtlänge des Strings)
(1) P Ö + Ö B = L
P Ö + Ö A S ich N ( θ ) = L

Differenzieren Sie nun den obigen Ausdruck nach der Zeit (wobei Sie wissen, dass OA konstant ist, und P Ö ˙ = u ):

P Ö ˙ Ö A C Ö S ( θ ) S ich N 2 ( θ ) θ ˙ = 0
(2) θ ˙ = S ich N 2 ( θ ) Ö A C Ö S ( θ ) u

Nehmen Sie nun die Gleichung 1 wieder setzen Ö B = A B C Ö S ( θ ) , und dann nach der Zeit differenzieren:

P Ö + A B C Ö S ( θ ) = L
P Ö ˙ + A B ˙ C Ö S ( θ ) + A B S ich N ( θ ) C Ö S 2 ( θ ) θ ˙ = 0

Jetzt, P Ö ˙ = u , A B ˙ = v Und A B = Ö A T A N ( θ ) . Somit:

u v C Ö S ( θ ) + Ö A S ich N ( θ ) T A N ( θ ) C Ö S 2 ( θ ) θ ˙ = 0
(3) u v C Ö S ( θ ) + Ö A C Ö S ( θ ) θ ˙ = 0

Nun, mit Gleichungen 2 Und 3 Eliminieren θ ˙ , erhalten wir die Beziehung:

u = v C Ö S ( θ )

Die Geschwindigkeit des Blocks nach oben entlang der Schnur wird auf das gesamte Seil übertragen. Wenn sich der Block also mit Geschwindigkeit nach oben bewegt v dann entlang des Seils ist die Komponente v cos ( θ ) was gleich der Geschwindigkeit des Seils ist u . Außerdem ist das erste zweifelhafte Diagramm 1 das richtige Diagramm. Denken Sie immer daran, dass die Komponente eines Vektors v kann niemals eine größere Größe haben als sie selbst. u ist die Komponente von v entlang der Schnur.
@sammygerbil: Ja, mir wurde klar, dass dies tatsächlich ein Duplikat dieser Frage ist. Während das Original wie ein Hausaufgabenproblem erscheint, bei dem sich der Benutzer keine Mühe gegeben hat, es zu lösen, ist meine Frage viel besser formuliert. Das Original wurde GESCHLOSSEN, da es als "Unklar" gekennzeichnet war. In diesem Fall sollte dies also nicht als Duplikat gezählt werden, oder?
Beide Fragen fragen dasselbe, und die andere war früher. Sie haben sich mehr Mühe gegeben, aber Ihre Frage wirft kein neues Problem auf. Ich habe nicht dafür gestimmt, es zu schließen, und bisher hat das auch niemand getan. Das Notieren von Duplikaten verbindet sie für zukünftige Suchen. Wenn Sie mit den hier bereitgestellten Antworten zufrieden sind, wählen Sie bitte eine aus.

Antworten (3)

Es gibt einen einfacheren Weg, um Ihre Lösung zu finden, rufen Sie einfach die Länge an Ö A gleich 1 (es ist konstant, also können wir alle Längen darauf skalieren) und die Länge AB gleich nennen z . Dann u gleich der negativen Änderungsrate der Länge der Hypotenuse ist Ö B (seit Ö B + Ö P konstant ist) und v gleich der negativen Änderungsrate von ist z . Die Hypotenuse ist die Quadratwurzel von 1 + z 2 , und seine negative Zeitableitung nach der Kettenregel ist cos ( θ ) mal D z / D T , das ist also v cos ( θ ) , und das entspricht u , und du bist fertig.

Der erste Schüler hat Recht, das ABC-Dreieck ist eine gute Projektion. Der zweite Student liegt falsch, weil das OAB-Dreieck keine gute Projektion ist, der Block bewegt sich nicht entlang OB. Der dritte Schüler liegt falsch, weil sich Geschwindigkeiten nicht so addieren – wenn Sie zwei gerade Seile an einem Block befestigt haben und beide Seile mit Geschwindigkeit ziehen u , der Block bewegt sich mit Geschwindigkeit u , nicht Geschwindigkeit 2 u .

Eigentlich können wir das jederzeit direkt sagen u = v cos ( θ ) . θ ist der momentane Winkel, und diese erste Methode ist nicht falsch. u ist einfach die Komponente von v entlang der Schnur. Die Länge des Strings muss konstant bleiben und somit gilt die Constraint-Relation.
Guter Punkt, das ist die einfachste Lösung. Ich habe mich falsch ausgedrückt, das Problem ist nicht die Änderungsrate von Theta, da die Projektion augenblicklich erfolgt. Der Schüler, der das ABC-Dreieck verwendet hat, hat Recht, da die tatsächliche Geschwindigkeit entlang AB verläuft und entlang OA projiziert werden kann. Der Schüler, der das OAB-Dreieck verwendet hat, liegt falsch, weil diese Lösung die tatsächliche Geschwindigkeit so behandelt, als ob sie entlang OA wäre und entlang AB projiziert werden kann. Es funktioniert nicht, so weiter zu projizieren, man müsste beide projizierten Komponenten einbeziehen.
Ich denke, Sie sollten Ihren Kommentar als Bearbeitung zur Antwort hinzufügen :). Verwenden Sie außerdem LaTeX beim Schreiben der mathematischen Symbole.
@KenG: Dein Kommentar verdeutlicht viel. Mein Zweifel war genau das, warum wir die Komponente von v entlang u nehmen und nicht u entlang v . Ihr Argument, dass "die tatsächliche Geschwindigkeit entlang AB verläuft und diese Geschwindigkeit daher entlang OA projiziert werden sollte und nicht umgekehrt", ist verdeutlichend. Dank dafür. +1
Vor einem Jahr hatte ich genau diesen Zweifel! Ich brauche etwas mehr Aufklärung. Sie sagen, dass es nicht funktioniert, so weiter zu projizieren, man müsste beide projizierten Komponenten einbeziehen. Könnten Sie bitte zeigen, wie oder welche beiden Komponenten addiert werden müssen, um die richtige Antwort zu erhalten, wenn wir das Dreieck OAB oder u als Geschwindigkeit verwenden und es in „welche alle Komponenten“ zerlegen, um die richtige Antwort zu erhalten. Es wäre sehr hilfreich, wenn Sie dieses Bit auch in die Antwort aufnehmen könnten. Danke
@shivams möchten Sie sich den obigen Kommentar von mir ansehen, da ich denke, Sie haben die Frage verstanden
@Shashaank: Ihre Frage ist nicht klar. Könntest du versuchen, es besser umzuformulieren?
@shivams Ich meinte, dass KenG sagte, dass "........ man müsste beide projizierten Komponenten einbeziehen" (in seinem letzten Kommentar, als er über das Brechen von OA spricht). Also wollte ich wissen, welche alle Komponenten werden nehmen wir dann?

Ich glaube, dieses Diagramm zeigt deutlich, wie sich der Ansatz Ihres ersten Schülers erklären lässt:

Geben Sie hier die Bildbeschreibung ein

An ähnlichen Dreiecken können Sie erkennen, dass das Seil mit zunehmender Entfernung kürzer wird u , bewegt sich die Last vertikal um eine Strecke u cos θ .

Was den Trugschluss des Ansatzes des zweiten Schülers betrifft: Während Geschwindigkeiten Vektoren sind und Vektoren summiert werden können, macht die Summierung nur Sinn, wenn Sie Bewegungen in verschiedenen Bezugssystemen betrachten. Wenn ich mich in einem Zug befinde, der sich mit Geschwindigkeit bewegt v , und ich werfe einen Ball mit Geschwindigkeit aus dem Fenster u , würde eine Person am Boden sehen, wie sich der Ball bewegt v + u . Aber wenn zwei Leute im Zug sehen, dass sich derselbe Ball bewegt u , Sie können nicht sagen "Nun, A sah eine Geschwindigkeit von u , und B sah eine Geschwindigkeit von u , also bewegt sich das Objekt um 2 u "...

+1 Für einen anderen und klareren Ansatz. Allerdings haben Sie eine andere genommen θ wie in der Frage angegeben. Dementsprechend sollte Ihre Antwort lauten u / S ich N ( θ ) .
@shivams - Entschuldigung, schlampig von mir ... danke für den Hinweis. Habe es jetzt behoben.
Irgendwie irrelevant, aber darf ich fragen, welche Software Sie für solche Diagramme/Illustrationen verwenden? Deine Darstellung sieht gut aus. Derzeit verwende ich IPE, das ich sehr liebe, aber ich bin immer auf der Suche.
@shivams - Ich verwende nur Powerpoint auf dem Mac ... Es hat eine anständige Reihe von Werkzeugen zum Ausrichten / Drehen usw. und funktioniert für die meisten meiner Diagramme.
Oh, in Ordnung. Ich vergesse immer die einfachen Werkzeuge für einen einfachen Job :)

Zweifel Nr. 1: Eine kurze Erklärung:

Im rechten Dreieck Ö A B , Seite Ö A ist konstant und cos θ = A B Ö B . Die Anwendung der Zeitableitung auf den Pathagorean Theorem für dieses Dreieck ergibt:

D D T ( ( Ö B ) 2 ) = D D T ( ( Ö A ) 2 + ( A B ) 2 ) 2 ( Ö B ) ( Ö B ˙ ) = 2 ( Ö A ) ( Ö A ˙ ) + 2 ( A B ) ( A B ˙ )
Vereinfachen und nutzen Ö A ˙ = 0 , Ö B ˙ = u Und A B ˙ = v , das wird u = v cos θ was das Ergebnis liefert
v = u cos θ

Zweifel Nr. 2: Der Faktor 2

Die Verwirrung ergibt sich aus der Art der im Problem angegebenen Anfangsbedingungen; Das Problem gibt die Geschwindigkeit von Punkten an P (linke feste Riemenscheibe) und Q (rechte feste Riemenscheibe) = u . Das Problem hätte stattdessen die angreifenden Kräfte angeben können P Und Q indem man eine Spannung gibt T in den Seilen und bittet Sie, für die Beschleunigung zu lösen M . Wenn dies der Fall gewesen wäre, wäre der Faktor von 2 würde in der Tat in der Antwort auftauchen (eine von Spannung auf der P Seite und eine vor Spannung auf der Q Seite), weil Kräfte angreifen M füge als Vektoren hinzu, während die Geschwindigkeit der Seile angehängt wird M unterlassen Sie.